LSAT and Law School Admissions Forum

Get expert LSAT preparation and law school admissions advice from PowerScore Test Preparation.

 Administrator
PowerScore Staff
  • PowerScore Staff
  • Posts: 8916
  • Joined: Feb 02, 2011
|
#85543
Complete Question Explanation

Assumption—Sufficient and Necessary. The correct answer choice is (B).

Answer choice (A):

Answer choice (B): This is the correct answer choice.

Answer choice (C):

Answer choice (D):

Answer choice (E):

This explanation is still in progress. Please post any questions below!
User avatar
 Michaelangilo
  • Posts: 5
  • Joined: Mar 26, 2021
|
#85855
Any explanation for this one? D looked good to me but I’m wrong and not sure why B is correct.
User avatar
 KelseyWoods
PowerScore Staff
  • PowerScore Staff
  • Posts: 1079
  • Joined: Jun 26, 2013
|
#85919
Hi Michaelangilo!

This stimulus contains conditional reasoning so we can diagram it:

Premise #1: Govt increases funding for civilian research :arrow: private patrons and industries believe civilian research is primarily govt responsibility
Premise #2: Private patrons and industries believe civilian research is primarily govt responsibility :arrow: private patrons and industries decrease contributions toward research
Conclusion: Depress overall level of funding for civilian scientific research :arrow: Govt increases funding for civilian research

With Assumption questions involving conditional reasoning, we are often looking for a gap in the conditional logic between the premises and the conclusion. Here, the author jumps from "private patrons and industries decrease contributions toward research" to "depress overall level of funding for civilian scientific research." So the author is assuming that the decrease in private funding that would happen after an increase in government funding would result in an overall decrease in funding. That's what answer choice (B) addresses: "Any increase in government funding would displace more private funding for civilian scientific research than it would provide." This answer choice tells us that the decrease in funding from private sources would be greater than the increase in funding from government sources, resulting in an overall decrease in funding.

Answer choice (D), on the other hand, states "Civilian scientific research cannot be conducted efficiently with more than one source of funding." This answer choice refers to efficiency of the research. But this is irrelevant to the argument. Nothing in the argument addresses the efficiency of the research. The argument is solely about funding amounts from different sources, not how efficiently the research can be conducted.

Hope this helps!

Best,
Kelsey
User avatar
 Michaelangilo
  • Posts: 5
  • Joined: Mar 26, 2021
|
#85940
Awesome, thanks Kelsey!! :)

Get the most out of your LSAT Prep Plus subscription.

Analyze and track your performance with our Testing and Analytics Package.